Math Question: Could someone explain how do we verify stokes theorem for the vector field F=zi + (2x+z)j + xk taken over the triangular surface S in the plane (x/1)+(y/2)+(z/3)=1 bounded by the planes x=0 y=0 and z=0. Take boundary of the above triangular surface as the path of the line integral. For the surface integral I got, integrate [x=0 - 1] , [y=0 , 2-2x] ( (1/14)*(54 - 38x - 27y) dx dy ) I got the y limit setting z=0 in the plane equation , ==> (y/2) + x = 1==> 2x+y=2 ==> y=2x-2 (Am i correct???) When working it the other way around using curl, i got Curl F = -i + 2k N = unt normal vector to the surface =k N.k = 2/7 (Curl F). N = -2/7 Fnally got the integrate [(-2/7) รท (2/7) dx dy] , limits : [x=0 - 1] , [y=0 , 2-2x] Finally I got -1 as the answer here. Could someone point out my mistakes please? Many thanks!

Asked By Ashley On 11/24/2019 01:51